Continuity of Linear Operator Between Hilbert SpacesNorm of adjoint operator in Hilbert spaceSesquilinear forms on Hilbert spacesGradient of inner product in Hilbert spaceDissipativity for Hilbert spacesA self-adjoint operator on a Hilbert spaceComplementary slackness in Hilbert spacesProof that every bounded linear operator between hilbert spaces has an adjoint.Proof explanation related to the operator matricesShowing that $exists x in H : |A(x)| = |A|_mathcalL$ if $H$ is Hilbert and $A in mathcalL_c(X,Y)$.Why is this operator symmetric? A question concerning a paper from Brezis and Crandall

Credit cards used everywhere in Singapore or Malaysia?

Sailing the cryptic seas

Provisioning profile doesn't include the application-identifier and keychain-access-groups entitlements

How to deal with a cynical class?

Look at your watch and tell me what time is it. vs Look at your watch and tell me what time it is

Interplanetary conflict, some disease destroys the ability to understand or appreciate music

Are all passive ability checks floors for active ability checks?

How do I hide Chekhov's Gun?

Does Mathematica reuse previous computations?

What options are left, if Britain cannot decide?

How to simplify this time periods definition interface?

Is it true that good novels will automatically sell themselves on Amazon (and so on) and there is no need for one to waste time promoting?

Why is the President allowed to veto a cancellation of emergency powers?

Who is flying the vertibirds?

Are there verbs that are neither telic, or atelic?

Do I need to be arrogant to get ahead?

Min function accepting varying number of arguments in C++17

Unexpected result from ArcLength

How to write cleanly even if my character uses expletive language?

Dice rolling probability game

It's a yearly task, alright

What's the meaning of “spike” in the context of “adrenaline spike”?

What did Alexander Pope mean by "Expletives their feeble Aid do join"?

Why doesn't using two cd commands in bash script execute the second command?



Continuity of Linear Operator Between Hilbert Spaces


Norm of adjoint operator in Hilbert spaceSesquilinear forms on Hilbert spacesGradient of inner product in Hilbert spaceDissipativity for Hilbert spacesA self-adjoint operator on a Hilbert spaceComplementary slackness in Hilbert spacesProof that every bounded linear operator between hilbert spaces has an adjoint.Proof explanation related to the operator matricesShowing that $exists x in H : |A(x)| = |A|_mathcalL$ if $H$ is Hilbert and $A in mathcalL_c(X,Y)$.Why is this operator symmetric? A question concerning a paper from Brezis and Crandall













2












$begingroup$



Note: Please do not give a solution; I am curious to understand why my solution is incorrect, and would prefer guidance to help me complete the question myself. Thank you.





Let $mathcalH$ be a Hilbert space, and suppose that $TintextHom(mathcalH,mathcalH)$. Suppose that there exists an operator $tildeT:mathcalHrightarrowmathcalH$ such that,
beginalign
langle Tx,yrangle =langle x,tildeTyrangle,
endalign

$forall x,yinmathcalH$. Show that $T$ is continuous.



My current solution is as follows:



Assume for all $delta>0$ there exists $n>NinmathbbN$ such that,
beginalign
|x_n-x|<delta.
endalign

Then,
beginalign
langle Tx_n-Tx,Tx_n-Txrangle &= |Tx_n-Tx|^2\
&leq|Tx_n-Tx|=|T(x_n-x)|\
&leq|T||x_n-x|rightarrow 0text as nrightarrowinfty.
endalign



What am I doing wrong? I notice I do not use the existence of $tildeT$.



Second Attempt:



Assume $langle x_n,xrangle rightarrow langle x,xrangle$ as $nrightarrowinfty$. Then, given $langle Tx,yrangle = langle x,tildeTyrangle$,
beginalign
langle Tx_n,yrangle &= langle x_n,tildeTyranglerightarrow_nrightarrowinftylangle x,tildeTyrangle=langle Tx,yrangle.
endalign

Therefore, $Tx_nrightarrow Tx$ as $nrightarrowinfty$.



Third Attempt:



Assume $|x_n-x|rightarrow 0$ as $nrightarrowinfty$. Then,
beginalign
langle Tx_n-Tx,Tx_n-Txrangle=langle x_n-x,x_n-xrangle=|x_n-x|^2.
endalign



By assumption $|x_n-x|^2rightarrow 0$ as $nrightarrowinfty$. Hence,
beginalign
langle Tx_n-Tx,Tx_n-Txrangle = |Tx_n-Tx|^2rightarrow 0text as nrightarrowinfty.
endalign

Therefore, $T$ is continuous.










share|cite|improve this question











$endgroup$







  • 2




    $begingroup$
    The last inequality basically implies that the norm of T is bounded or that it is continuous
    $endgroup$
    – Andres Mejia
    4 hours ago






  • 1




    $begingroup$
    Comment on the second attempt: you showed that $Tx_n to Tx$ weakly, not in norm. Off-topic comment: I admire your tenacity. Keep trying!
    $endgroup$
    – Umberto P.
    3 hours ago











  • $begingroup$
    Thank you. Do you have a hint?
    $endgroup$
    – Jack
    3 hours ago










  • $begingroup$
    Third attempt made. Although not sure if this holds either.
    $endgroup$
    – Jack
    3 hours ago















2












$begingroup$



Note: Please do not give a solution; I am curious to understand why my solution is incorrect, and would prefer guidance to help me complete the question myself. Thank you.





Let $mathcalH$ be a Hilbert space, and suppose that $TintextHom(mathcalH,mathcalH)$. Suppose that there exists an operator $tildeT:mathcalHrightarrowmathcalH$ such that,
beginalign
langle Tx,yrangle =langle x,tildeTyrangle,
endalign

$forall x,yinmathcalH$. Show that $T$ is continuous.



My current solution is as follows:



Assume for all $delta>0$ there exists $n>NinmathbbN$ such that,
beginalign
|x_n-x|<delta.
endalign

Then,
beginalign
langle Tx_n-Tx,Tx_n-Txrangle &= |Tx_n-Tx|^2\
&leq|Tx_n-Tx|=|T(x_n-x)|\
&leq|T||x_n-x|rightarrow 0text as nrightarrowinfty.
endalign



What am I doing wrong? I notice I do not use the existence of $tildeT$.



Second Attempt:



Assume $langle x_n,xrangle rightarrow langle x,xrangle$ as $nrightarrowinfty$. Then, given $langle Tx,yrangle = langle x,tildeTyrangle$,
beginalign
langle Tx_n,yrangle &= langle x_n,tildeTyranglerightarrow_nrightarrowinftylangle x,tildeTyrangle=langle Tx,yrangle.
endalign

Therefore, $Tx_nrightarrow Tx$ as $nrightarrowinfty$.



Third Attempt:



Assume $|x_n-x|rightarrow 0$ as $nrightarrowinfty$. Then,
beginalign
langle Tx_n-Tx,Tx_n-Txrangle=langle x_n-x,x_n-xrangle=|x_n-x|^2.
endalign



By assumption $|x_n-x|^2rightarrow 0$ as $nrightarrowinfty$. Hence,
beginalign
langle Tx_n-Tx,Tx_n-Txrangle = |Tx_n-Tx|^2rightarrow 0text as nrightarrowinfty.
endalign

Therefore, $T$ is continuous.










share|cite|improve this question











$endgroup$







  • 2




    $begingroup$
    The last inequality basically implies that the norm of T is bounded or that it is continuous
    $endgroup$
    – Andres Mejia
    4 hours ago






  • 1




    $begingroup$
    Comment on the second attempt: you showed that $Tx_n to Tx$ weakly, not in norm. Off-topic comment: I admire your tenacity. Keep trying!
    $endgroup$
    – Umberto P.
    3 hours ago











  • $begingroup$
    Thank you. Do you have a hint?
    $endgroup$
    – Jack
    3 hours ago










  • $begingroup$
    Third attempt made. Although not sure if this holds either.
    $endgroup$
    – Jack
    3 hours ago













2












2








2


1



$begingroup$



Note: Please do not give a solution; I am curious to understand why my solution is incorrect, and would prefer guidance to help me complete the question myself. Thank you.





Let $mathcalH$ be a Hilbert space, and suppose that $TintextHom(mathcalH,mathcalH)$. Suppose that there exists an operator $tildeT:mathcalHrightarrowmathcalH$ such that,
beginalign
langle Tx,yrangle =langle x,tildeTyrangle,
endalign

$forall x,yinmathcalH$. Show that $T$ is continuous.



My current solution is as follows:



Assume for all $delta>0$ there exists $n>NinmathbbN$ such that,
beginalign
|x_n-x|<delta.
endalign

Then,
beginalign
langle Tx_n-Tx,Tx_n-Txrangle &= |Tx_n-Tx|^2\
&leq|Tx_n-Tx|=|T(x_n-x)|\
&leq|T||x_n-x|rightarrow 0text as nrightarrowinfty.
endalign



What am I doing wrong? I notice I do not use the existence of $tildeT$.



Second Attempt:



Assume $langle x_n,xrangle rightarrow langle x,xrangle$ as $nrightarrowinfty$. Then, given $langle Tx,yrangle = langle x,tildeTyrangle$,
beginalign
langle Tx_n,yrangle &= langle x_n,tildeTyranglerightarrow_nrightarrowinftylangle x,tildeTyrangle=langle Tx,yrangle.
endalign

Therefore, $Tx_nrightarrow Tx$ as $nrightarrowinfty$.



Third Attempt:



Assume $|x_n-x|rightarrow 0$ as $nrightarrowinfty$. Then,
beginalign
langle Tx_n-Tx,Tx_n-Txrangle=langle x_n-x,x_n-xrangle=|x_n-x|^2.
endalign



By assumption $|x_n-x|^2rightarrow 0$ as $nrightarrowinfty$. Hence,
beginalign
langle Tx_n-Tx,Tx_n-Txrangle = |Tx_n-Tx|^2rightarrow 0text as nrightarrowinfty.
endalign

Therefore, $T$ is continuous.










share|cite|improve this question











$endgroup$





Note: Please do not give a solution; I am curious to understand why my solution is incorrect, and would prefer guidance to help me complete the question myself. Thank you.





Let $mathcalH$ be a Hilbert space, and suppose that $TintextHom(mathcalH,mathcalH)$. Suppose that there exists an operator $tildeT:mathcalHrightarrowmathcalH$ such that,
beginalign
langle Tx,yrangle =langle x,tildeTyrangle,
endalign

$forall x,yinmathcalH$. Show that $T$ is continuous.



My current solution is as follows:



Assume for all $delta>0$ there exists $n>NinmathbbN$ such that,
beginalign
|x_n-x|<delta.
endalign

Then,
beginalign
langle Tx_n-Tx,Tx_n-Txrangle &= |Tx_n-Tx|^2\
&leq|Tx_n-Tx|=|T(x_n-x)|\
&leq|T||x_n-x|rightarrow 0text as nrightarrowinfty.
endalign



What am I doing wrong? I notice I do not use the existence of $tildeT$.



Second Attempt:



Assume $langle x_n,xrangle rightarrow langle x,xrangle$ as $nrightarrowinfty$. Then, given $langle Tx,yrangle = langle x,tildeTyrangle$,
beginalign
langle Tx_n,yrangle &= langle x_n,tildeTyranglerightarrow_nrightarrowinftylangle x,tildeTyrangle=langle Tx,yrangle.
endalign

Therefore, $Tx_nrightarrow Tx$ as $nrightarrowinfty$.



Third Attempt:



Assume $|x_n-x|rightarrow 0$ as $nrightarrowinfty$. Then,
beginalign
langle Tx_n-Tx,Tx_n-Txrangle=langle x_n-x,x_n-xrangle=|x_n-x|^2.
endalign



By assumption $|x_n-x|^2rightarrow 0$ as $nrightarrowinfty$. Hence,
beginalign
langle Tx_n-Tx,Tx_n-Txrangle = |Tx_n-Tx|^2rightarrow 0text as nrightarrowinfty.
endalign

Therefore, $T$ is continuous.







functional-analysis continuity hilbert-spaces






share|cite|improve this question















share|cite|improve this question













share|cite|improve this question




share|cite|improve this question








edited 3 hours ago







Jack

















asked 5 hours ago









JackJack

887




887







  • 2




    $begingroup$
    The last inequality basically implies that the norm of T is bounded or that it is continuous
    $endgroup$
    – Andres Mejia
    4 hours ago






  • 1




    $begingroup$
    Comment on the second attempt: you showed that $Tx_n to Tx$ weakly, not in norm. Off-topic comment: I admire your tenacity. Keep trying!
    $endgroup$
    – Umberto P.
    3 hours ago











  • $begingroup$
    Thank you. Do you have a hint?
    $endgroup$
    – Jack
    3 hours ago










  • $begingroup$
    Third attempt made. Although not sure if this holds either.
    $endgroup$
    – Jack
    3 hours ago












  • 2




    $begingroup$
    The last inequality basically implies that the norm of T is bounded or that it is continuous
    $endgroup$
    – Andres Mejia
    4 hours ago






  • 1




    $begingroup$
    Comment on the second attempt: you showed that $Tx_n to Tx$ weakly, not in norm. Off-topic comment: I admire your tenacity. Keep trying!
    $endgroup$
    – Umberto P.
    3 hours ago











  • $begingroup$
    Thank you. Do you have a hint?
    $endgroup$
    – Jack
    3 hours ago










  • $begingroup$
    Third attempt made. Although not sure if this holds either.
    $endgroup$
    – Jack
    3 hours ago







2




2




$begingroup$
The last inequality basically implies that the norm of T is bounded or that it is continuous
$endgroup$
– Andres Mejia
4 hours ago




$begingroup$
The last inequality basically implies that the norm of T is bounded or that it is continuous
$endgroup$
– Andres Mejia
4 hours ago




1




1




$begingroup$
Comment on the second attempt: you showed that $Tx_n to Tx$ weakly, not in norm. Off-topic comment: I admire your tenacity. Keep trying!
$endgroup$
– Umberto P.
3 hours ago





$begingroup$
Comment on the second attempt: you showed that $Tx_n to Tx$ weakly, not in norm. Off-topic comment: I admire your tenacity. Keep trying!
$endgroup$
– Umberto P.
3 hours ago













$begingroup$
Thank you. Do you have a hint?
$endgroup$
– Jack
3 hours ago




$begingroup$
Thank you. Do you have a hint?
$endgroup$
– Jack
3 hours ago












$begingroup$
Third attempt made. Although not sure if this holds either.
$endgroup$
– Jack
3 hours ago




$begingroup$
Third attempt made. Although not sure if this holds either.
$endgroup$
– Jack
3 hours ago










1 Answer
1






active

oldest

votes


















4












$begingroup$

The problem is that we can't assume that $T$ has a finite norm. Before we add that condition about having an adjoint map $tildeT$, we're simply assuming that $T$ is a linear map.



In fact, a linear map between normed vector spaces is continuous if and only if it has a finite operator norm. You assumed the statement we were trying to prove.



Second attempt: The assumption here should have been that $x_nto x$, as in the others. Then, yes, $langle Tx_n,yrangle to langle Tx,yrangle$ for each $y$. This is real progress. But, as stated in the comments, it's weak convergence rather than convergence in norm. Not quite there.



Third attempt: No, $langle Tu,Turangle$ is not equal to $langle u,urangle$ - it's equal to $langle u,tildeTTurangle$, and you don't know what $tildeTT$ does. This is not helpful.



All, right, lets go back to the attempt that made some progress. Are you familiar with the uniform boundedness principle? One consequence of that theorem is that any sequence of points in a Hilbert space that converges weakly is bounded. Can we use this to ensure that $T$ is a bounded operator?






share|cite|improve this answer











$endgroup$












  • $begingroup$
    So is the idea for me to use $tildeT$ to cancel out the operator norm in my final inequality?
    $endgroup$
    – Jack
    4 hours ago










  • $begingroup$
    @Jack no, that won't rescue the proof.
    $endgroup$
    – Umberto P.
    3 hours ago










  • $begingroup$
    New proof attempt. Please check if you can.
    $endgroup$
    – Jack
    3 hours ago










Your Answer





StackExchange.ifUsing("editor", function ()
return StackExchange.using("mathjaxEditing", function ()
StackExchange.MarkdownEditor.creationCallbacks.add(function (editor, postfix)
StackExchange.mathjaxEditing.prepareWmdForMathJax(editor, postfix, [["$", "$"], ["\\(","\\)"]]);
);
);
, "mathjax-editing");

StackExchange.ready(function()
var channelOptions =
tags: "".split(" "),
id: "69"
;
initTagRenderer("".split(" "), "".split(" "), channelOptions);

StackExchange.using("externalEditor", function()
// Have to fire editor after snippets, if snippets enabled
if (StackExchange.settings.snippets.snippetsEnabled)
StackExchange.using("snippets", function()
createEditor();
);

else
createEditor();

);

function createEditor()
StackExchange.prepareEditor(
heartbeatType: 'answer',
autoActivateHeartbeat: false,
convertImagesToLinks: true,
noModals: true,
showLowRepImageUploadWarning: true,
reputationToPostImages: 10,
bindNavPrevention: true,
postfix: "",
imageUploader:
brandingHtml: "Powered by u003ca class="icon-imgur-white" href="https://imgur.com/"u003eu003c/au003e",
contentPolicyHtml: "User contributions licensed under u003ca href="https://creativecommons.org/licenses/by-sa/3.0/"u003ecc by-sa 3.0 with attribution requiredu003c/au003e u003ca href="https://stackoverflow.com/legal/content-policy"u003e(content policy)u003c/au003e",
allowUrls: true
,
noCode: true, onDemand: true,
discardSelector: ".discard-answer"
,immediatelyShowMarkdownHelp:true
);



);













draft saved

draft discarded


















StackExchange.ready(
function ()
StackExchange.openid.initPostLogin('.new-post-login', 'https%3a%2f%2fmath.stackexchange.com%2fquestions%2f3149977%2fcontinuity-of-linear-operator-between-hilbert-spaces%23new-answer', 'question_page');

);

Post as a guest















Required, but never shown

























1 Answer
1






active

oldest

votes








1 Answer
1






active

oldest

votes









active

oldest

votes






active

oldest

votes









4












$begingroup$

The problem is that we can't assume that $T$ has a finite norm. Before we add that condition about having an adjoint map $tildeT$, we're simply assuming that $T$ is a linear map.



In fact, a linear map between normed vector spaces is continuous if and only if it has a finite operator norm. You assumed the statement we were trying to prove.



Second attempt: The assumption here should have been that $x_nto x$, as in the others. Then, yes, $langle Tx_n,yrangle to langle Tx,yrangle$ for each $y$. This is real progress. But, as stated in the comments, it's weak convergence rather than convergence in norm. Not quite there.



Third attempt: No, $langle Tu,Turangle$ is not equal to $langle u,urangle$ - it's equal to $langle u,tildeTTurangle$, and you don't know what $tildeTT$ does. This is not helpful.



All, right, lets go back to the attempt that made some progress. Are you familiar with the uniform boundedness principle? One consequence of that theorem is that any sequence of points in a Hilbert space that converges weakly is bounded. Can we use this to ensure that $T$ is a bounded operator?






share|cite|improve this answer











$endgroup$












  • $begingroup$
    So is the idea for me to use $tildeT$ to cancel out the operator norm in my final inequality?
    $endgroup$
    – Jack
    4 hours ago










  • $begingroup$
    @Jack no, that won't rescue the proof.
    $endgroup$
    – Umberto P.
    3 hours ago










  • $begingroup$
    New proof attempt. Please check if you can.
    $endgroup$
    – Jack
    3 hours ago















4












$begingroup$

The problem is that we can't assume that $T$ has a finite norm. Before we add that condition about having an adjoint map $tildeT$, we're simply assuming that $T$ is a linear map.



In fact, a linear map between normed vector spaces is continuous if and only if it has a finite operator norm. You assumed the statement we were trying to prove.



Second attempt: The assumption here should have been that $x_nto x$, as in the others. Then, yes, $langle Tx_n,yrangle to langle Tx,yrangle$ for each $y$. This is real progress. But, as stated in the comments, it's weak convergence rather than convergence in norm. Not quite there.



Third attempt: No, $langle Tu,Turangle$ is not equal to $langle u,urangle$ - it's equal to $langle u,tildeTTurangle$, and you don't know what $tildeTT$ does. This is not helpful.



All, right, lets go back to the attempt that made some progress. Are you familiar with the uniform boundedness principle? One consequence of that theorem is that any sequence of points in a Hilbert space that converges weakly is bounded. Can we use this to ensure that $T$ is a bounded operator?






share|cite|improve this answer











$endgroup$












  • $begingroup$
    So is the idea for me to use $tildeT$ to cancel out the operator norm in my final inequality?
    $endgroup$
    – Jack
    4 hours ago










  • $begingroup$
    @Jack no, that won't rescue the proof.
    $endgroup$
    – Umberto P.
    3 hours ago










  • $begingroup$
    New proof attempt. Please check if you can.
    $endgroup$
    – Jack
    3 hours ago













4












4








4





$begingroup$

The problem is that we can't assume that $T$ has a finite norm. Before we add that condition about having an adjoint map $tildeT$, we're simply assuming that $T$ is a linear map.



In fact, a linear map between normed vector spaces is continuous if and only if it has a finite operator norm. You assumed the statement we were trying to prove.



Second attempt: The assumption here should have been that $x_nto x$, as in the others. Then, yes, $langle Tx_n,yrangle to langle Tx,yrangle$ for each $y$. This is real progress. But, as stated in the comments, it's weak convergence rather than convergence in norm. Not quite there.



Third attempt: No, $langle Tu,Turangle$ is not equal to $langle u,urangle$ - it's equal to $langle u,tildeTTurangle$, and you don't know what $tildeTT$ does. This is not helpful.



All, right, lets go back to the attempt that made some progress. Are you familiar with the uniform boundedness principle? One consequence of that theorem is that any sequence of points in a Hilbert space that converges weakly is bounded. Can we use this to ensure that $T$ is a bounded operator?






share|cite|improve this answer











$endgroup$



The problem is that we can't assume that $T$ has a finite norm. Before we add that condition about having an adjoint map $tildeT$, we're simply assuming that $T$ is a linear map.



In fact, a linear map between normed vector spaces is continuous if and only if it has a finite operator norm. You assumed the statement we were trying to prove.



Second attempt: The assumption here should have been that $x_nto x$, as in the others. Then, yes, $langle Tx_n,yrangle to langle Tx,yrangle$ for each $y$. This is real progress. But, as stated in the comments, it's weak convergence rather than convergence in norm. Not quite there.



Third attempt: No, $langle Tu,Turangle$ is not equal to $langle u,urangle$ - it's equal to $langle u,tildeTTurangle$, and you don't know what $tildeTT$ does. This is not helpful.



All, right, lets go back to the attempt that made some progress. Are you familiar with the uniform boundedness principle? One consequence of that theorem is that any sequence of points in a Hilbert space that converges weakly is bounded. Can we use this to ensure that $T$ is a bounded operator?







share|cite|improve this answer














share|cite|improve this answer



share|cite|improve this answer








edited 1 hour ago

























answered 4 hours ago









jmerryjmerry

14.3k1629




14.3k1629











  • $begingroup$
    So is the idea for me to use $tildeT$ to cancel out the operator norm in my final inequality?
    $endgroup$
    – Jack
    4 hours ago










  • $begingroup$
    @Jack no, that won't rescue the proof.
    $endgroup$
    – Umberto P.
    3 hours ago










  • $begingroup$
    New proof attempt. Please check if you can.
    $endgroup$
    – Jack
    3 hours ago
















  • $begingroup$
    So is the idea for me to use $tildeT$ to cancel out the operator norm in my final inequality?
    $endgroup$
    – Jack
    4 hours ago










  • $begingroup$
    @Jack no, that won't rescue the proof.
    $endgroup$
    – Umberto P.
    3 hours ago










  • $begingroup$
    New proof attempt. Please check if you can.
    $endgroup$
    – Jack
    3 hours ago















$begingroup$
So is the idea for me to use $tildeT$ to cancel out the operator norm in my final inequality?
$endgroup$
– Jack
4 hours ago




$begingroup$
So is the idea for me to use $tildeT$ to cancel out the operator norm in my final inequality?
$endgroup$
– Jack
4 hours ago












$begingroup$
@Jack no, that won't rescue the proof.
$endgroup$
– Umberto P.
3 hours ago




$begingroup$
@Jack no, that won't rescue the proof.
$endgroup$
– Umberto P.
3 hours ago












$begingroup$
New proof attempt. Please check if you can.
$endgroup$
– Jack
3 hours ago




$begingroup$
New proof attempt. Please check if you can.
$endgroup$
– Jack
3 hours ago

















draft saved

draft discarded
















































Thanks for contributing an answer to Mathematics Stack Exchange!


  • Please be sure to answer the question. Provide details and share your research!

But avoid


  • Asking for help, clarification, or responding to other answers.

  • Making statements based on opinion; back them up with references or personal experience.

Use MathJax to format equations. MathJax reference.


To learn more, see our tips on writing great answers.




draft saved


draft discarded














StackExchange.ready(
function ()
StackExchange.openid.initPostLogin('.new-post-login', 'https%3a%2f%2fmath.stackexchange.com%2fquestions%2f3149977%2fcontinuity-of-linear-operator-between-hilbert-spaces%23new-answer', 'question_page');

);

Post as a guest















Required, but never shown





















































Required, but never shown














Required, but never shown












Required, but never shown







Required, but never shown

































Required, but never shown














Required, but never shown












Required, but never shown







Required, but never shown







Popular posts from this blog

Identifying “long and narrow” polygons in with PostGISlength and width of polygonWhy postgis st_overlaps reports Qgis' “avoid intersections” generated polygon as overlapping with others?Adjusting polygons to boundary and filling holesDrawing polygons with fixed area?How to remove spikes in Polygons with PostGISDeleting sliver polygons after difference operation in QGIS?Snapping boundaries in PostGISSplit polygon into parts adding attributes based on underlying polygon in QGISSplitting overlap between polygons and assign to nearest polygon using PostGIS?Expanding polygons and clipping at midpoint?Removing Intersection of Buffers in Same Layers

Masuk log Menu navigasi

อาณาจักร (ชีววิทยา) ดูเพิ่ม อ้างอิง รายการเลือกการนำทาง10.1086/39456810.5962/bhl.title.447410.1126/science.163.3863.150576276010.1007/BF01796092408502"Phylogenetic structure of the prokaryotic domain: the primary kingdoms"10.1073/pnas.74.11.5088432104270744"Towards a natural system of organisms: proposal for the domains Archaea, Bacteria, and Eucarya"1990PNAS...87.4576W10.1073/pnas.87.12.4576541592112744PubMedJump the queueexpand by handPubMedJump the queueexpand by handPubMedJump the queueexpand by hand"A revised six-kingdom system of life"10.1111/j.1469-185X.1998.tb00030.x9809012"Only six kingdoms of life"10.1098/rspb.2004.2705169172415306349"Kingdoms Protozoa and Chromista and the eozoan root of the eukaryotic tree"10.1098/rsbl.2009.0948288006020031978เพิ่มข้อมูล